Prove that the standard basis vectors span R^2

Click For Summary
SUMMARY

The discussion centers on the concept of vector spaces, specifically addressing why R^2 is considered a vector space over the field of real numbers (R) rather than the field of rational numbers (Q). It is established that while standard basis vectors e_1 and e_2 (or i and j) span R^2, attempting to define a basis over Q leads to complications, including the necessity of the axiom of choice due to the uncountably infinite nature of such a basis. This distinction is crucial for understanding the properties and dimensions of vector spaces.

PREREQUISITES
  • Understanding of vector spaces and their properties
  • Familiarity with standard basis vectors in R^2
  • Knowledge of fields, specifically R and Q
  • Basic comprehension of the axiom of choice in set theory
NEXT STEPS
  • Study the properties of vector spaces over different fields
  • Learn about the implications of the axiom of choice in linear algebra
  • Explore the concept of basis and dimension in vector spaces
  • Investigate the differences between countable and uncountable sets
USEFUL FOR

Students of linear algebra, mathematicians exploring vector spaces, and educators teaching the fundamentals of vector spaces and basis concepts.

Mathematicsresear
Messages
66
Reaction score
0

Homework Statement


I know how to approach this problem; however, I'm just confused as to why we consider that R^2 is a vector space over the field R, and not Q or any other field for this question?

Standard basis vectors: e_1, e_2 or i,j
 
Physics news on Phys.org
Mathematicsresear said:

Homework Statement


I know how to approach this problem; however, I'm just confused as to why we consider that R^2 is a vector space over the field R, and not Q or any other field for this question?

Standard basis vectors: e_1, e_2 or i,j

You will have difficulty writing down a basis of ##\mathbb{R}^2## over the field ##\mathbb{Q}##: such a basis is uncountably infinite and you need the axiom of choice to show it exists.
 
Question: A clock's minute hand has length 4 and its hour hand has length 3. What is the distance between the tips at the moment when it is increasing most rapidly?(Putnam Exam Question) Answer: Making assumption that both the hands moves at constant angular velocities, the answer is ## \sqrt{7} .## But don't you think this assumption is somewhat doubtful and wrong?

Similar threads

  • · Replies 17 ·
Replies
17
Views
4K
Replies
15
Views
2K
  • · Replies 3 ·
Replies
3
Views
2K
  • · Replies 9 ·
Replies
9
Views
4K
  • · Replies 16 ·
Replies
16
Views
3K
  • · Replies 4 ·
Replies
4
Views
3K
  • · Replies 9 ·
Replies
9
Views
3K
  • · Replies 2 ·
Replies
2
Views
2K
Replies
7
Views
2K
  • · Replies 4 ·
Replies
4
Views
2K